Question

Five years ago, as part of a plan to encourage citizens of Levaska to increase the amount of money they put into savings, Levaska’s government introduced special savings accounts in which up to $3,000 a year can be saved with no tax due on the interest unless money is withdrawn before the account holder reaches the age of sixty-five. Millions of dollars have accumulated in the special accounts, so the government’s plan is obviously working.

Which of the following, if true, most seriously weakens the argument?

(This question is from Official Guide. Therefore, because of copyrights, the complete question cannot be copied here. The question can be accessed at GMAT Club)

Solution

Understanding the Passage.

Five years ago, as part of a plan to encourage citizens of Levaska to increase the amount of money they put into savings,

The plan’s objective is: to encourage citizens of Levaska to increase their savings.

Levaska’s government introduced special savings accounts in which up to $3,000 a year can be saved with no tax due on the interest unless money is withdrawn before the account holder reaches the age of sixty-five.

The plan: Launch of SSA (Special Savings Accounts) – if the money is not withdrawn before the age of 65, no tax on the interest amount for up to a principal amount of $3000 per year.

Millions of dollars have accumulated in the special accounts, so the government’s plan is obviously working.

Since millions of dollars have accumulated in these SSAs, the author concludes that the government’s plan is working.

What is the government’s plan?

Launch SSA

What is the government’s goal?

To encourage citizens of Levaska to increase their savings.

When can we say that the government’s plan is working?

When the plan is leading to the goal. Or, in other words, when the launch of SSA is encouraging citizens of Levaska to increase their savings.

The Gap

Can we be sure that the government’s plan is working, given that millions of dollars have accumulated in these SSAs?

If not, we have gaps in the argument.

Here are the gaps I can think of:

  1. Money in these SSAs need not mean more savings. For example, the citizens could have diverted their savings from other modes of savings to these accounts. 
  2. More savings need not mean more savings of the citizens of Levaska. For example, there are probably non-citizens of Levaska living in Levaska. Maybe, these are the people who invested in these SSAs. In such a case, the plan (to increase citizens’ savings) is not working.

There can be more gaps in the argument. Can you think of any? I’d like to hear from you.

The Evaluation

(A) Incorrect.

This option has NO IMPACT on the argument.

Let me create a stronger version of this option:

A’: A substantial number of Levaskans have withdrawn a substantial amount of the money they had invested in the special accounts.

What is the impact of A’ on the argument?

No impact.

That 90% of the people have withdrawn 90% of the money they had invested in the special accounts also has NO IMPACT on the argument. Why? Because it’s given in the argument that millions of dollars are lying in these accounts. Thus, the savings seem to have increased. So, the plan (to increase savings) seems to be working.

The fact that savings have not increased to the level they could have increased has NO IMPACT on our plan.

(B) Incorrect

This option says that a particular category of workers (who already save money in long-term tax-free accounts that are offered through their workplace) cannot use SSAs.

This option has NO IMPACT on the argument.

Let me create a stronger version of this option.

B’: Most of the workers in Levaska cannot take advantage of the special savings accounts introduced by the government.

B’ also has NO IMPACT on the argument.

Even if 90% of the workers in Levaska cannot take advantage of these SSAs, the argument will not be impacted. Why? Because it’s given in the argument that millions of dollars are lying in these accounts. Thus, the savings seem to have increased. So, the plan (to increase savings) seems to be working.

Given that the argument has already given that there is an amount lying in SSAs, we’re not concerned with what proportion of the population has contributed to that amount (whether it’s 1% of the population or 90% of the population). Irrespective of the proportion of the population that contributed to the amounts in SSAs, the savings seem to have increased. Thus, this option is irrelevant.

(C) Incorrect

This option says that the tax rate applicable on interest earned from money in the REGULAR savings account depends on the income bracket of the account holder. (Please note that this option is about regular savings accounts. For special savings accounts, we’re given that interest is anyway tax-free if the money is not withdrawn before the account holder reaches the age of 65.)

Thus, some people would need to pay higher taxes than others.

This option has NO IMPACT on the argument.

Whether people have different or the same tax rates on their interest income is irrelevant to the argument.

(D) Correct.

This option is around the first gap we identified. If people are just transferring their savings from another mode of savings to SSAs, then money in SSAs doesn’t mean more savings. Thus, this option creates doubt on the conclusion that the plan is working.

Please remember that the question asks to find an option that weakens or creates doubt on the argument. We’re not necessarily looking for an option that demolishes the argument.

(E) Incorrect

This option says that many economists (who now claim that the government’s plan has been successful) criticized the plan when the plan was introduced.

Whether these people criticized the plan at the time of its introduction or not has NO IMPACT on our argument.

Leave a comment

Leave a Reply

Share this:

Like this:

Like Loading...

Discover more from GMAT with CJ

Subscribe now to keep reading and get access to the full archive.

Continue reading